Quanten.de Diskussionsforum  

Zur?ck   Quanten.de Diskussionsforum > Quantenmechanik, Relativitätstheorie und der ganze Rest.

Hinweise

Quantenmechanik, Relativitätstheorie und der ganze Rest. Wenn Sie Themen diskutieren wollen, die mehr als Schulkenntnisse voraussetzen, sind Sie hier richtig. Keine Angst, ein Physikstudium ist nicht Voraussetzung, aber man sollte sich schon eingehender mit Physik beschäftigt haben.

Antwort
 
Themen-Optionen Ansicht
  #11  
Alt 20.07.16, 16:55
Plankton Plankton ist offline
Guru
 
Registriert seit: 02.01.2015
Beitr?ge: 915
Standard AW: Fragen zu einer Publikation (Measuring Quantum Coherence with Entanglement)

Zitat:
Zitat von inside Beitrag anzeigen
Es beantwortet Deine letzte Frage.
Nein tut er nicht! Es geht hier im Thread um Kohärenz speziell in der QM und exakt um eine Erklärung zu der Arbeit aus dem Eingangspost! Nicht um "zwei Sinuswellen" die interferieren! Also bitte keine Allgemeinplätze mehr.
http://arxiv.org/pdf/1502.05876v4.pdf

2 Fragen:
1. Kann ich mir dann vorstellen das "Coherence" eines Systems bedeutet, z.B. dass es in einer Superposition ist aus Spin UP/DOWN x,y,z, oder/und anderer QM-Eigenschaften?
2. [Wenn man das vorherige so ansehen kann,] was bedeutet dann "Incoherent"?

Ge?ndert von Plankton (20.07.16 um 16:57 Uhr)
Mit Zitat antworten
  #12  
Alt 21.07.16, 14:32
Plankton Plankton ist offline
Guru
 
Registriert seit: 02.01.2015
Beitr?ge: 915
Standard AW: Fragen zu einer Publikation (Measuring Quantum Coherence with Entanglement)

Gibt es keinen Experten hier der was dazu sagen kann?

Will nur wissen ob ein inkohärenter Zustand ein fester Zustand eines QM-Systems ist z.B. aus Spin Down y, oder Spin Up x?
(Wenn ein kohärenter Zustand eine Superposition ist z.B. aus Spin Up/Down y, und/oder Spin Up/Down x.)
Mit Zitat antworten
  #13  
Alt 01.08.16, 09:00
Plankton Plankton ist offline
Guru
 
Registriert seit: 02.01.2015
Beitr?ge: 915
Standard AW: Fragen zu einer Publikation (Measuring Quantum Coherence with Entanglement)

Leider, leider hat mich der Thread nicht wirklich schlau gemacht.
Werde wohl mein (Dangerous ) Halbwissen nun auf den Stand aktualisieren:

Kohärenter/Coherent State = Superposition z.B. aus Spin UP/DOWN x
Inkohärenter/Incoherent State = keine Superpostion, nur ein Zustand aus z.B. Spin UP x, oder Spin DOWN x
Mit Zitat antworten
  #14  
Alt 29.08.16, 21:36
Plankton Plankton ist offline
Guru
 
Registriert seit: 02.01.2015
Beitr?ge: 915
Standard AW: Fragen zu einer Publikation (Measuring Quantum Coherence with Entanglement)

Hier in dem Papier wird nochmal besonders auf diese Arbeiten eingegangen:
http://quantumcorrelations.weebly.co...ations_cde.pdf

The maximum entanglement 𝐸 between the system 𝑆 and the initially incoherent ancilla 𝐴 created by incoherent operations quantifies the initial coherence 𝐶𝐸 in the system state 𝜌𝑆

Ich verstehe das so, dass die maximal mögliche Superposition von System S quantifiziert ist über die maximale mögliche Verschränkung von System S und System A.

BTW: Falls jemand Interesse und Lust hat, würde ich mich freuen wenn jemand auf diesen Satz eingehen kann mit einem Beispiel:
Bipartite coherence: a state is incoherent if it is diagonal in a local product basis
Mit Zitat antworten
  #15  
Alt 01.09.16, 10:34
Plankton Plankton ist offline
Guru
 
Registriert seit: 02.01.2015
Beitr?ge: 915
Daumen hoch Irgendjemand weiß das doch!!!

Der ganze Thread kann auf die Frage reduziert werden:

Was ist ein Zustand der Diagonal in der Referenzbasis ist?
Formel dazu:

Q: http://quantumcorrelations.weebly.co...erencetalk.pdf
Angeh?ngte Grafiken
Dateityp: jpg Screenshot (123).jpg (13,4 KB, 3x aufgerufen)
Mit Zitat antworten
  #16  
Alt 01.09.16, 11:10
Hawkwind Hawkwind ist offline
Singularität
 
Registriert seit: 22.07.2010
Ort: Rabenstein, Niederösterreich
Beitr?ge: 3.057
Standard AW: Irgendjemand weiß das doch!!!

Zitat:
Zitat von Plankton Beitrag anzeigen
Der ganze Thread kann auf die Frage reduziert werden:

Was ist ein Zustand der Diagonal in der Referenzbasis ist?
Formel dazu:

Q: http://quantumcorrelations.weebly.co...erencetalk.pdf
Das sind Zustände, die sich in ein Produkt der Unter-Zustände separieren lassen, z.B. ein 2-Teilchen-Zustand, der einfach ein äußeres Produkt der 1-Teilchen-Zustände ist ==> unverschränkt.

Hintergrund dazu gibt es z.B. hier:
http://www.physik.uni-siegen.de/quan...chtematrix.pdf

Siehe besonders Seite 19.
Mit Zitat antworten
  #17  
Alt 01.09.16, 12:53
Plankton Plankton ist offline
Guru
 
Registriert seit: 02.01.2015
Beitr?ge: 915
Standard AW: Irgendjemand weiß das doch!!!

DANKE!
Zitat:
Zitat von Hawkwind Beitrag anzeigen
Das sind Zustände, die sich in ein Produkt der Unter-Zustände separieren lassen, z.B. ein 2-Teilchen-Zustand, der einfach ein äußeres Produkt der 1-Teilchen-Zustände ist ==> unverschränkt.

Hintergrund dazu gibt es z.B. hier:
http://www.physik.uni-siegen.de/quan...chtematrix.pdf

Siehe besonders Seite 19.
Hier könnte ich dann das Teilsystem beschreiben (im Gegensatz zur Verschränkung)?
Mit Zitat antworten
  #18  
Alt 01.09.16, 13:33
Plankton Plankton ist offline
Guru
 
Registriert seit: 02.01.2015
Beitr?ge: 915
Standard AW: Fragen zu einer Publikation (Measuring Quantum Coherence with Entanglement)

Puhhhhhh.... ich glaub ich muss mich nochmal grundsätzlich mit der ganzen Thematik auseinandersetzen.

Ge?ndert von Plankton (01.09.16 um 15:58 Uhr) Grund: Post von vorher irrelevant
Mit Zitat antworten
  #19  
Alt 13.09.16, 00:15
Plankton Plankton ist offline
Guru
 
Registriert seit: 02.01.2015
Beitr?ge: 915
Blinzeln Two sides of the same coin?

http://arxiv.org/pdf/1502.05876v4.pdf

Wäre es unzulässig das so zu reduzieren?

Kohärenter Zustand + inkohärenter Zustand (quasi "klassisch") = Verschränkung
bzw.
maximale Kohärenz = maximale Verschränkung
Mit Zitat antworten
  #20  
Alt 13.09.16, 15:58
Hawkwind Hawkwind ist offline
Singularität
 
Registriert seit: 22.07.2010
Ort: Rabenstein, Niederösterreich
Beitr?ge: 3.057
Standard AW: Two sides of the same coin?

Zitat:
Zitat von Plankton Beitrag anzeigen
Kohärenter Zustand + inkohärenter Zustand (quasi "klassisch") = Verschränkung
Nein, das passt m.E. nicht. Ein verschränkter Zustand ist ein reiner Quantenzustand, der aus 2 oder mehr Subsystemen(Quanten) besteht (meist an verschiedene Orten), und zwar derart, dass die Wellenfunktion des Gesamtsystems sich nicht als Produkt der Wellenfunktionen der Subsysteme beschreiben lässt.
Mit Zitat antworten
Antwort

Lesezeichen

Themen-Optionen
Ansicht

Forumregeln
Es ist Ihnen nicht erlaubt, neue Themen zu verfassen.
Es ist Ihnen nicht erlaubt, auf Beitr?ge zu antworten.
Es ist Ihnen nicht erlaubt, Anh?nge hochzuladen.
Es ist Ihnen nicht erlaubt, Ihre Beitr?ge zu bearbeiten.

BB-Code ist an.
Smileys sind an.
[IMG] Code ist an.
HTML-Code ist aus.

Gehe zu


Alle Zeitangaben in WEZ +1. Es ist jetzt 14:48 Uhr.


Powered by vBulletin® Version 3.8.8 (Deutsch)
Copyright ©2000 - 2024, vBulletin Solutions, Inc.
ScienceUp - Dr. Günter Sturm